Last visit was: 27 Apr 2024, 21:19 It is currently 27 Apr 2024, 21:19

Close
GMAT Club Daily Prep
Thank you for using the timer - this advanced tool can estimate your performance and suggest more practice questions. We have subscribed you to Daily Prep Questions via email.

Customized
for You

we will pick new questions that match your level based on your Timer History

Track
Your Progress

every week, we’ll send you an estimated GMAT score based on your performance

Practice
Pays

we will pick new questions that match your level based on your Timer History
Not interested in getting valuable practice questions and articles delivered to your email? No problem, unsubscribe here.
Close
Request Expert Reply
Confirm Cancel
SORT BY:
Date
Tags:
Show Tags
Hide Tags
Math Expert
Joined: 02 Sep 2009
Posts: 92959
Own Kudos [?]: 619508 [2]
Given Kudos: 81611
Send PM
Current Student
Joined: 18 Aug 2016
Posts: 531
Own Kudos [?]: 577 [0]
Given Kudos: 198
Concentration: Strategy, Technology
GMAT 1: 630 Q47 V29
GMAT 2: 740 Q51 V38
Send PM
avatar
Intern
Intern
Joined: 08 Jun 2017
Posts: 8
Own Kudos [?]: 1 [0]
Given Kudos: 4
Send PM
Math Expert
Joined: 02 Sep 2009
Posts: 92959
Own Kudos [?]: 619508 [0]
Given Kudos: 81611
Send PM
Re: In the diagram below, what is the measure of ∠BCD? [#permalink]
Expert Reply
mynamegoeson wrote:
Bunuel wrote:
In the diagram below, what is the measure of ∠BCD?


(1) AB is parallel to DE.
(2) ∠ABC = 550.


Attachment:
2017-07-25_1016.png


Hi Bunuel

is ∠ABC = 550 correct..i suppose it will be ∠ABC = 55 Degree


It's 55°. Edited. Thank you.
Intern
Intern
Joined: 09 Jan 2014
Posts: 9
Own Kudos [?]: 1 [0]
Given Kudos: 67
Location: India
Schools: Oxford"19 (A)
GMAT 1: 660 Q47 V34
GPA: 3.75
Send PM
Re: In the diagram below, what is the measure of ∠BCD? [#permalink]
Is it C??

Since both statements would be required to measure the angle BCD
Manager
Manager
Joined: 22 Nov 2016
Posts: 205
Own Kudos [?]: 267 [0]
Given Kudos: 50
Concentration: Leadership, Strategy
Send PM
In the diagram below, what is the measure of ∠BCD? [#permalink]
Let \(\angle BCD = x\)

Using statement 1:

From the new diagram attached, you can extend BC to intersect DE at a new point, say F.
The angle formed, i.e \(\angle CFD = \angle ABC = y\)

In triangle CFD, \(\angle CDF = 90 \angle FCD = 180 -x\)
180 - x + y + 90 = 180
x-y = 90

Hence 1 is insufficient.

Statement 2 gives us \(\angle ABC\) but we dont know if AB and DE are ||

Using 1 and 2, we can find \(\angle BCD = 180 - 35 = 145\)
Attachments

slice1.png
slice1.png [ 10.69 KiB | Viewed 3371 times ]

GMAT Club Bot
In the diagram below, what is the measure of ∠BCD? [#permalink]
Moderator:
Math Expert
92959 posts

Powered by phpBB © phpBB Group | Emoji artwork provided by EmojiOne